1answer.
Ask question
Login Signup
Ask question
All categories
  • English
  • Mathematics
  • Social Studies
  • Business
  • History
  • Health
  • Geography
  • Biology
  • Physics
  • Chemistry
  • Computers and Technology
  • Arts
  • World Languages
  • Spanish
  • French
  • German
  • Advanced Placement (AP)
  • SAT
  • Medicine
  • Law
  • Engineering
Mkey [24]
3 years ago
7

Gretchen made a paper cone to hold a gift for a friend. The paper cone was

Mathematics
1 answer:
AVprozaik [17]3 years ago
6 0

Answer: 803.84 --> 803.8

Step-by-step explanation:

4^4=16x3.14=50.24*16=803.84

You might be interested in
If the radius of the circle is 9, find the area of the shaded region. Round your
zmey [24]

Answer:

we dont know the shaded area...

Step-by-step explanation:

6 0
3 years ago
Say that Thailand has a workforce of 35,709,487 people, each of whom earns an average salary of (equivalent USD) $8,630 annually
marysya [2.9K]

The income tax rate must be 29%.

Given that,

Thailand has a workforce of 35,709,487 people, each of whom earns an average salary of (equivalent USD) $8,630 annually.

If the Thai government wants to raise $70 billion in tax revenue,

We have to find,

The approximately where should it set the income tax rate?

According to the question,

Thailand has a workforce of 35,709,487 people, each of whom earns an average salary of (equivalent USD) $8,630 annually.

\rm Total \ income = Number \ of \ people \times Income \ per \ person\\\\Total \ income = 35,709,487 \times 8,630\\\\Total \ income = 308,172,872,810

If the Thai government wants to raise $70 billion in tax revenue,

Then,

The income tax rate is,

\rm Tax\  rate = \dfrac{70,000,000,000,}{308,172,872,810}\\\\Tax \ rate = 0.29\\\\Tax \ rate = 29 \ percent

Hence, The income tax rate must be 29%.

For more details refer to the link given below.

brainly.com/question/16016455

6 0
2 years ago
Write an equation of the line in slope-intercept form: y-intercept of 2, x-intercept of 4
zvonat [6]

Answer:

\sf y=-\dfrac12x+2

Step-by-step explanation:

y-intercept is when x = 0, so (0, 2)

x-intercept is when y = 0, so (4, 0)

\sf slope=\dfrac{change\:in\:y}{change\:in\:x}=\dfrac{0-2}{4-0}=-\dfrac12

Slope-intercept form of linear equation:  \sf y=mx+b

(where m is the slope and b is the y-intercept)

Given:

  • \sf m=-\dfrac12
  • b = 2

\sf \implies y=-\dfrac12x+2

3 0
2 years ago
You are paid $7.25/hour. You work 40 hours/week for 2 weeks. Your involuntary deductions are FICA (7.65%), federal withholding (
Vedmedyk [2.9K]

Answer:

$413.83

Step-by-step explanation:

Your earnings for the two weeks are

7.25(40)(2) = 580

The deductions for FICA will be

0.0765(580) = 44.37

The deductions for federal withholding will be

0.14(580) = 81.2

The deductions for state withholding will be

0.07(580) = 40.6

This makes your realized income

580-(44.37+81.2+40.6) = 580-166.17 = 413.83

7 0
3 years ago
Read 2 more answers
What is the greatest value of 2,463.9051
RoseWind [281]

Hey There @Bre18016,

The answer is \boxed{2}

The greatest value of 2,463.9051 would be the thousands place (2) simply as it is the biggest number out of the other places.

For instance, if we had the number 300, 3 would be the greatest value.

Or let's say we had 10,000 the 1 would be the greatest value.

Furthermore, you could look at the first digit in the entire number to deter mine the greatest value.

6 0
2 years ago
Other questions:
  • Let f(x)=(x+2)2.
    13·2 answers
  • 3x-5y=9whats the answer
    7·1 answer
  • Dylan bought 3 identical shirts online for a total cost of $71.83 including a flat rate of $7.99 for shipping. Drag numbers to c
    13·1 answer
  • I NEED HELP ASAP<br><br>are the time/distance rates equivalent? ​
    6·1 answer
  • 3a+3b,for a=2 and b=4
    15·1 answer
  • Joseph is shopping for school supplies. He can buy a box of 24 pens for $4.32 or a box of 36 pens for $5.76. Which is a better b
    8·1 answer
  • What is the fraction of 44 out of 160?
    7·1 answer
  • While standing on a pier, Lynette saw a balloon floating 20 feet above her and a fish swimming 4 feet below the surface of the w
    14·2 answers
  • PLEASE HELP ME ASAP!!!!!!!!!!!!!!!
    12·1 answer
  • Simplify the expression. Write your answer using decimals.
    11·2 answers
Add answer
Login
Not registered? Fast signup
Signup
Login Signup
Ask question!